Difference between revisions of "2013 AMC 12A Problems/Problem 10"

(Created page with "Note that <math>\frac{1}{11} = 0.\overline{09}</math>. Dividing by 3 gives <math>\frac{1}{33} = 0.\overline{03}</math>, and dividing by 9 gives <math>\frac{1}{99} = 0.\overline{...")
 
Line 1: Line 1:
 +
== Problem==
 +
 +
Let <math>S</math> be the set of positive integers <math>n</math> for which <math>\tfrac{1}{n}</math> has the repeating decimal representation <math>0.\overline{ab} = 0.ababab\cdots,</math> with <math>a</math> and <math>b</math> different digits.  What is the sum of the elements of <math>S</math>?
 +
 +
<math> \textbf{(A)}\ 11\qquad\textbf{(B)}\ 44\qquad\textbf{(C)}\ 110\qquad\textbf{(D)}\ 143\qquad\textbf{(E)}\ 155\qquad </math>
 +
 +
==Solution==
 +
 
Note that <math>\frac{1}{11} = 0.\overline{09}</math>.
 
Note that <math>\frac{1}{11} = 0.\overline{09}</math>.
  
Line 8: Line 16:
  
 
The answer must be at least <math>143</math>, but cannot be <math>155</math> since no <math>n \le 12</math> other than <math>11</math> satisfies the conditions, so the answer is <math>143</math>.
 
The answer must be at least <math>143</math>, but cannot be <math>155</math> since no <math>n \le 12</math> other than <math>11</math> satisfies the conditions, so the answer is <math>143</math>.
 +
 +
== See also ==
 +
{{AMC12 box|year=2013|ab=A|num-b=9|num-a=11}}

Revision as of 18:37, 22 February 2013

Problem

Let $S$ be the set of positive integers $n$ for which $\tfrac{1}{n}$ has the repeating decimal representation $0.\overline{ab} = 0.ababab\cdots,$ with $a$ and $b$ different digits. What is the sum of the elements of $S$?

$\textbf{(A)}\ 11\qquad\textbf{(B)}\ 44\qquad\textbf{(C)}\ 110\qquad\textbf{(D)}\ 143\qquad\textbf{(E)}\ 155\qquad$

Solution

Note that $\frac{1}{11} = 0.\overline{09}$.

Dividing by 3 gives $\frac{1}{33} = 0.\overline{03}$, and dividing by 9 gives $\frac{1}{99} = 0.\overline{01}$.

$S = \{11, 33, 99\}$

$11 + 33 + 99 = 143$

The answer must be at least $143$, but cannot be $155$ since no $n \le 12$ other than $11$ satisfies the conditions, so the answer is $143$.

See also

2013 AMC 12A (ProblemsAnswer KeyResources)
Preceded by
Problem 9
Followed by
Problem 11
1 2 3 4 5 6 7 8 9 10 11 12 13 14 15 16 17 18 19 20 21 22 23 24 25
All AMC 12 Problems and Solutions